INTEGRALBOT BETA-DIGAMMA INTEGRAL

Today we will show the following result that appears in this twitter post


\int_{0}^{\infty} \frac{x}{1+x^{3}} \ln \left(1+\frac{1}{x^{3}}\right) d x=\frac{\pi \ln (3)}{\sqrt{3}}-\frac{\pi^{2}}{9}


We will start by proving two lemmas involving the Beta and Digamma functions.



Lemma 1:

\int_0^\infty \frac{x^{s-1}\ln(x)}{1+x}\,dx=\Gamma(s)\Gamma(1-s)\left(\psi(s)-\psi(1-s) \right)(1)


Recall the Beta function


I(s)=\int_0^\infty \frac{x^{s-1}}{1+x}\,dx=\Gamma(s)\Gamma(1-s)(2)


If we differentiate (2) w.r. to s we obtain


\begin{aligned}
I^\prime(s)&=\int_0^\infty \frac{x^{s-1}\ln(x)}{1+x}\,dx=\frac{d}{ds}\left[\Gamma(s)\Gamma(1-s)\right]\\
&=\Gamma^\prime(s)\Gamma(1-s)-\Gamma(s)\Gamma^\prime(1-s)\\
&=\Gamma(s)\Gamma(1-s)\psi(s)-\Gamma(s)\Gamma(1-s)\psi(1-s)\\
&=\Gamma(s)\Gamma(1-s)\left(\psi(s)-\psi(1-s) \right) \qquad \blacksquare
\end{aligned}


Lemma 2:


\int_0^1 x^{a-1}(1-x)^{b-1}\ln(x)\,dx=\frac{\Gamma(a)\Gamma(b)}{\Gamma(a+b)}\left(\psi(a)-\psi(a+b) \right)
(3)


Recall the Beta function


I(a)=\int_0^1 x^{a-1}(1-x)^{b-1}\,dx=\frac{\Gamma(a)\Gamma(b)}{\Gamma(a+b)}(4)


If we differentiate (4) w.r. to a we obtain


\begin{aligned}
I^\prime(a)&=\int_0^1 x^{a-1}(1-x)^{b-1}\ln(x)\,dx=\frac{d}{da}\left[\frac{\Gamma(a)\Gamma(b)}{\Gamma(a+b)}\right]\\
&=\frac{\Gamma^\prime(a)\Gamma(b)\Gamma(a+b)-\Gamma(a)\Gamma(b)\Gamma^\prime(a+b)}{\Gamma^2(a+b)}\\
&=\frac{\psi(a)\Gamma(a)\Gamma(b)\Gamma(a+b)-\Gamma(a)\Gamma(b)\Gamma(a+b)\psi(a+b)}{\Gamma^2(a+b)}\\
&=\frac{\Gamma(a)\Gamma(b)}{\Gamma(a+b)}\left(\psi(a)-\psi(a+b) \right) \qquad \blacksquare
\end{aligned}


Now let´s evaluate the integral:



\begin{aligned}
\int_{0}^{\infty} \frac{x}{1+x^{3}} \ln \left(1+\frac{1}{x^{3}}\right) \,d x&=\int_{0}^{\infty} \frac{x}{1+x^{3}} \ln \left(\frac{1+x^{3}}{x^{3}}\right) \,d x\\
&=\int_{0}^{\infty} \frac{x}{1+x^{3}} \ln \left(1+x^{3}\right) \,d x-3\int_{0}^{\infty} \frac{x}{1+x^{3}} \ln \left(x\right) \,d x\\
&=J-3K
\end{aligned}



\begin{aligned}
J&=\int_{0}^{\infty} \frac{x}{1+x^{3}} \ln \left(1+x^{3}\right) \,d x\\
&=\frac13\int_1^\infty \frac{(x-1)^{\frac23-1}\ln(x)}{x}\,dx \qquad \left(1+x^3 \to x \right)\\
&=-\frac13 \int_0^1x^{-\frac23}(1-x)^{\frac23-1}\ln(x),dx\\
&=-\frac13 \frac{\Gamma\left(\frac13\right)\Gamma\left(\frac23\right)}{\Gamma(1)}\left(\psi\left(\frac13\right)-\psi(1) \right) \qquad \left(\text{by lemma 2} \right)\\
&=-\frac{\pi}{3}\frac{1}{\sin\left(\frac{\pi}{3} \right)}\left(-\frac{\pi}{2 \sqrt{3}}-\frac{3 \ln(3)}{2}-\gamma+ \gamma\right)\\
&=\frac{\pi^2}{9}+\frac{\pi \ln(3)}{\sqrt{3}} \qquad \blacksquare
\end{aligned}

For the proof of the special values of the Digamma function see this post.



\begin{aligned}
K&=\int_{0}^{\infty} \frac{x}{1+x^{3}} \ln \left(x\right) \,d x\\
&=\frac13 \int_0^\infty \frac{x^{\frac23-1}\ln(x^{1/3})}{1+x}\,dx \qquad \left(x^3 \to x \right)\\
&=\frac19 \int_0^\infty \frac{x^{\frac23-1}\ln(x^{1/3})}{1+x}\,dx\\
&=\frac19\Gamma\left(\frac23\right)\Gamma\left(\frac13\right)\left(\psi\left(\frac23\right)-\psi\left(\frac13\right) \right)\\
&=-\frac{\pi}{9\sin\left(\frac{2\pi}{3} \right)}\left(\psi\left(\frac13\right)-\psi\left(\frac23\right) \right)\\
&=-\frac{2\pi}{9\sqrt{3}}\left(-\frac{\pi}{\sqrt{3}}\right)\\
&=\frac{2 \pi^2}{27} \qquad \blacksquare
\end{aligned}


Plugging the values of J and K back in the original integral we obtain:


\begin{aligned}
\int_{0}^{\infty} \frac{x}{1+x^{3}} \ln \left(1+\frac{1}{x^{3}}\right) \,d x
&=J-3K\\
&=\frac{\pi^2}{9}+\frac{\pi \ln(3)}{\sqrt{3}}-3\frac{2 \pi^2}{27}\\
&=\frac{\pi \ln (3)}{\sqrt{3}}-\frac{\pi^{2}}{9} \qquad \blacksquare
\end{aligned}

Comments

Popular posts from this blog